Difference between revisions of "2021 Fall AMC 10A Problems/Problem 3"
MRENTHUSIASM (talk | contribs) m |
(→Solution 1 (Inequality)) |
||
(10 intermediate revisions by 6 users not shown) | |||
Line 22: | Line 22: | ||
By an overestimation <math>\pi\approx\frac{22}{7},</math> we have <math>4\pi<\frac{88}{7},</math> or <math>\frac{81}{4\pi}>6\frac{39}{88}.</math> | By an overestimation <math>\pi\approx\frac{22}{7},</math> we have <math>4\pi<\frac{88}{7},</math> or <math>\frac{81}{4\pi}>6\frac{39}{88}.</math> | ||
− | Together, we get <cmath>6\frac{39}{88} < \frac{81}{4\pi} < 6\frac34,</cmath> | + | Together, we get <cmath>6 < 6\frac{39}{88} < \frac{81}{4\pi} < 6\frac34 < 7,</cmath> |
from which <math>\left\lfloor\frac{81}{4\pi}\right\rfloor=\boxed{\textbf{(D) }6}.</math> | from which <math>\left\lfloor\frac{81}{4\pi}\right\rfloor=\boxed{\textbf{(D) }6}.</math> | ||
Line 33: | Line 33: | ||
Approximating with <math>\pi\approx3,</math> we have <math>\frac{81}{4\pi}\approx6\frac34.</math> Since <math>\pi</math> is about <math>5\%</math> greater than <math>3,</math> it is safe to claim that <math>\left\lfloor\frac{81}{4\pi}\right\rfloor=\boxed{\textbf{(D) }6}.</math> | Approximating with <math>\pi\approx3,</math> we have <math>\frac{81}{4\pi}\approx6\frac34.</math> Since <math>\pi</math> is about <math>5\%</math> greater than <math>3,</math> it is safe to claim that <math>\left\lfloor\frac{81}{4\pi}\right\rfloor=\boxed{\textbf{(D) }6}.</math> | ||
− | ~Arcticturn | + | ~Arcticturn ~MRENTHUSIASM |
+ | |||
+ | ==Video Solution== | ||
+ | https://youtu.be/0WrcZI_z9IE | ||
+ | |||
+ | ~savannahsolver | ||
+ | |||
+ | ==Video Solution== | ||
+ | https://youtu.be/qOiO45IsE54 | ||
+ | |||
+ | ~Charles3829 | ||
+ | |||
+ | ==Video Solution by TheBeautyofMath== | ||
+ | https://youtu.be/o98vGHAUYjM?t=158 | ||
+ | |||
+ | ~IceMatrix | ||
+ | |||
+ | ==Video Solution (HOW TO THINK CREATIVELY!!!)== | ||
+ | https://youtu.be/VcGR0GVqKBI | ||
+ | |||
+ | ~Education, the Study of Everything | ||
==See Also== | ==See Also== | ||
{{AMC10 box|year=2021 Fall|ab=A|num-b=2|num-a=4}} | {{AMC10 box|year=2021 Fall|ab=A|num-b=2|num-a=4}} | ||
{{MAA Notice}} | {{MAA Notice}} |
Latest revision as of 16:14, 19 October 2023
Contents
Problem
What is the maximum number of balls of clay of radius that can completely fit inside a cube of side length assuming the balls can be reshaped but not compressed before they are packed in the cube?
Solution 1 (Inequality)
The volume of the cube is and the volume of a clay ball is
Since the balls can be reshaped but not compressed, the maximum number of balls that can completely fit inside a cube is Approximating with we have or We simplify to get from which
~NH14 ~MRENTHUSIASM
Solution 2 (Inequality)
As shown in Solution 1, we conclude that the maximum number of balls that can completely fit inside a cube is
By an underestimation we have or
By an overestimation we have or
Together, we get from which
~MRENTHUSIASM
Solution 3 (Approximation)
As shown in Solution 1, we conclude that the maximum number of balls that can completely fit inside a cube is
Approximating with we have Since is about greater than it is safe to claim that
~Arcticturn ~MRENTHUSIASM
Video Solution
~savannahsolver
Video Solution
~Charles3829
Video Solution by TheBeautyofMath
https://youtu.be/o98vGHAUYjM?t=158
~IceMatrix
Video Solution (HOW TO THINK CREATIVELY!!!)
~Education, the Study of Everything
See Also
2021 Fall AMC 10A (Problems • Answer Key • Resources) | ||
Preceded by Problem 2 |
Followed by Problem 4 | |
1 • 2 • 3 • 4 • 5 • 6 • 7 • 8 • 9 • 10 • 11 • 12 • 13 • 14 • 15 • 16 • 17 • 18 • 19 • 20 • 21 • 22 • 23 • 24 • 25 | ||
All AMC 10 Problems and Solutions |
The problems on this page are copyrighted by the Mathematical Association of America's American Mathematics Competitions.